Are the HBL derivability conditions necessary for Gödel's incompleteness theorems? (For Löb's theorem?)












1












$begingroup$


I am currently working with 'proof theory and logical complexity', a monograph on proof theory.



In the exercises of the first chapter one was asked to prove Löb's theorem (https://en.wikipedia.org/wiki/L%C3%B6b%27s_theorem) and two related/similar statements.
(The first chapter essentially gives the setup to prove and proves both incompleteness theorems and some results around them.)



Searching the internet I came across the Hilbert-Bernays-Löb 'derivability conditions' (https://en.wikipedia.org/wiki/Hilbert%E2%80%93Bernays_provability_conditions), that are - apparently - necessary to prove Löb's theorem.



The author never mentions them explicitly, and while the first is very plausible and the second not unplausible, I read that the third's proof is tedious and difficult.



The author never gives a proof for them, and I don't recall that one would have been needed, so I ask myself whether it is possible to prove the incompleteness theorems without them, or if they follow easily from some result, especially since Löb's theorem is in this case an exercise, and would be quite hard if the author intends the student to have the idea and the proof of the derivability conditions all by himself. Or maybe Löb's theorem is itself provable without them?



Thanks,



Ettore



PS: I added the question also on mathoverflow:
https://mathoverflow.net/questions/319402/are-the-hbl-derivability-conditions-necessary-for-g%C3%B6dels-incompleteness-theorem










share|cite|improve this question











$endgroup$








  • 1




    $begingroup$
    Not strictly needed; G's original proof does not use them. They are used for a "abstract" (i.e. generalized) version of the theorem. See e.g. B.Buldt, The Scope of Gödel’s First Incompleteness Theorem (2014).
    $endgroup$
    – Mauro ALLEGRANZA
    Dec 6 '18 at 10:58
















1












$begingroup$


I am currently working with 'proof theory and logical complexity', a monograph on proof theory.



In the exercises of the first chapter one was asked to prove Löb's theorem (https://en.wikipedia.org/wiki/L%C3%B6b%27s_theorem) and two related/similar statements.
(The first chapter essentially gives the setup to prove and proves both incompleteness theorems and some results around them.)



Searching the internet I came across the Hilbert-Bernays-Löb 'derivability conditions' (https://en.wikipedia.org/wiki/Hilbert%E2%80%93Bernays_provability_conditions), that are - apparently - necessary to prove Löb's theorem.



The author never mentions them explicitly, and while the first is very plausible and the second not unplausible, I read that the third's proof is tedious and difficult.



The author never gives a proof for them, and I don't recall that one would have been needed, so I ask myself whether it is possible to prove the incompleteness theorems without them, or if they follow easily from some result, especially since Löb's theorem is in this case an exercise, and would be quite hard if the author intends the student to have the idea and the proof of the derivability conditions all by himself. Or maybe Löb's theorem is itself provable without them?



Thanks,



Ettore



PS: I added the question also on mathoverflow:
https://mathoverflow.net/questions/319402/are-the-hbl-derivability-conditions-necessary-for-g%C3%B6dels-incompleteness-theorem










share|cite|improve this question











$endgroup$








  • 1




    $begingroup$
    Not strictly needed; G's original proof does not use them. They are used for a "abstract" (i.e. generalized) version of the theorem. See e.g. B.Buldt, The Scope of Gödel’s First Incompleteness Theorem (2014).
    $endgroup$
    – Mauro ALLEGRANZA
    Dec 6 '18 at 10:58














1












1








1





$begingroup$


I am currently working with 'proof theory and logical complexity', a monograph on proof theory.



In the exercises of the first chapter one was asked to prove Löb's theorem (https://en.wikipedia.org/wiki/L%C3%B6b%27s_theorem) and two related/similar statements.
(The first chapter essentially gives the setup to prove and proves both incompleteness theorems and some results around them.)



Searching the internet I came across the Hilbert-Bernays-Löb 'derivability conditions' (https://en.wikipedia.org/wiki/Hilbert%E2%80%93Bernays_provability_conditions), that are - apparently - necessary to prove Löb's theorem.



The author never mentions them explicitly, and while the first is very plausible and the second not unplausible, I read that the third's proof is tedious and difficult.



The author never gives a proof for them, and I don't recall that one would have been needed, so I ask myself whether it is possible to prove the incompleteness theorems without them, or if they follow easily from some result, especially since Löb's theorem is in this case an exercise, and would be quite hard if the author intends the student to have the idea and the proof of the derivability conditions all by himself. Or maybe Löb's theorem is itself provable without them?



Thanks,



Ettore



PS: I added the question also on mathoverflow:
https://mathoverflow.net/questions/319402/are-the-hbl-derivability-conditions-necessary-for-g%C3%B6dels-incompleteness-theorem










share|cite|improve this question











$endgroup$




I am currently working with 'proof theory and logical complexity', a monograph on proof theory.



In the exercises of the first chapter one was asked to prove Löb's theorem (https://en.wikipedia.org/wiki/L%C3%B6b%27s_theorem) and two related/similar statements.
(The first chapter essentially gives the setup to prove and proves both incompleteness theorems and some results around them.)



Searching the internet I came across the Hilbert-Bernays-Löb 'derivability conditions' (https://en.wikipedia.org/wiki/Hilbert%E2%80%93Bernays_provability_conditions), that are - apparently - necessary to prove Löb's theorem.



The author never mentions them explicitly, and while the first is very plausible and the second not unplausible, I read that the third's proof is tedious and difficult.



The author never gives a proof for them, and I don't recall that one would have been needed, so I ask myself whether it is possible to prove the incompleteness theorems without them, or if they follow easily from some result, especially since Löb's theorem is in this case an exercise, and would be quite hard if the author intends the student to have the idea and the proof of the derivability conditions all by himself. Or maybe Löb's theorem is itself provable without them?



Thanks,



Ettore



PS: I added the question also on mathoverflow:
https://mathoverflow.net/questions/319402/are-the-hbl-derivability-conditions-necessary-for-g%C3%B6dels-incompleteness-theorem







logic proof-theory incompleteness meta-math provability






share|cite|improve this question















share|cite|improve this question













share|cite|improve this question




share|cite|improve this question








edited Dec 24 '18 at 10:33







Ettore

















asked Dec 6 '18 at 10:39









EttoreEttore

989




989








  • 1




    $begingroup$
    Not strictly needed; G's original proof does not use them. They are used for a "abstract" (i.e. generalized) version of the theorem. See e.g. B.Buldt, The Scope of Gödel’s First Incompleteness Theorem (2014).
    $endgroup$
    – Mauro ALLEGRANZA
    Dec 6 '18 at 10:58














  • 1




    $begingroup$
    Not strictly needed; G's original proof does not use them. They are used for a "abstract" (i.e. generalized) version of the theorem. See e.g. B.Buldt, The Scope of Gödel’s First Incompleteness Theorem (2014).
    $endgroup$
    – Mauro ALLEGRANZA
    Dec 6 '18 at 10:58








1




1




$begingroup$
Not strictly needed; G's original proof does not use them. They are used for a "abstract" (i.e. generalized) version of the theorem. See e.g. B.Buldt, The Scope of Gödel’s First Incompleteness Theorem (2014).
$endgroup$
– Mauro ALLEGRANZA
Dec 6 '18 at 10:58




$begingroup$
Not strictly needed; G's original proof does not use them. They are used for a "abstract" (i.e. generalized) version of the theorem. See e.g. B.Buldt, The Scope of Gödel’s First Incompleteness Theorem (2014).
$endgroup$
– Mauro ALLEGRANZA
Dec 6 '18 at 10:58










0






active

oldest

votes











Your Answer





StackExchange.ifUsing("editor", function () {
return StackExchange.using("mathjaxEditing", function () {
StackExchange.MarkdownEditor.creationCallbacks.add(function (editor, postfix) {
StackExchange.mathjaxEditing.prepareWmdForMathJax(editor, postfix, [["$", "$"], ["\\(","\\)"]]);
});
});
}, "mathjax-editing");

StackExchange.ready(function() {
var channelOptions = {
tags: "".split(" "),
id: "69"
};
initTagRenderer("".split(" "), "".split(" "), channelOptions);

StackExchange.using("externalEditor", function() {
// Have to fire editor after snippets, if snippets enabled
if (StackExchange.settings.snippets.snippetsEnabled) {
StackExchange.using("snippets", function() {
createEditor();
});
}
else {
createEditor();
}
});

function createEditor() {
StackExchange.prepareEditor({
heartbeatType: 'answer',
autoActivateHeartbeat: false,
convertImagesToLinks: true,
noModals: true,
showLowRepImageUploadWarning: true,
reputationToPostImages: 10,
bindNavPrevention: true,
postfix: "",
imageUploader: {
brandingHtml: "Powered by u003ca class="icon-imgur-white" href="https://imgur.com/"u003eu003c/au003e",
contentPolicyHtml: "User contributions licensed under u003ca href="https://creativecommons.org/licenses/by-sa/3.0/"u003ecc by-sa 3.0 with attribution requiredu003c/au003e u003ca href="https://stackoverflow.com/legal/content-policy"u003e(content policy)u003c/au003e",
allowUrls: true
},
noCode: true, onDemand: true,
discardSelector: ".discard-answer"
,immediatelyShowMarkdownHelp:true
});


}
});














draft saved

draft discarded


















StackExchange.ready(
function () {
StackExchange.openid.initPostLogin('.new-post-login', 'https%3a%2f%2fmath.stackexchange.com%2fquestions%2f3028327%2fare-the-hbl-derivability-conditions-necessary-for-g%25c3%25b6dels-incompleteness-theorem%23new-answer', 'question_page');
}
);

Post as a guest















Required, but never shown

























0






active

oldest

votes








0






active

oldest

votes









active

oldest

votes






active

oldest

votes
















draft saved

draft discarded




















































Thanks for contributing an answer to Mathematics Stack Exchange!


  • Please be sure to answer the question. Provide details and share your research!

But avoid



  • Asking for help, clarification, or responding to other answers.

  • Making statements based on opinion; back them up with references or personal experience.


Use MathJax to format equations. MathJax reference.


To learn more, see our tips on writing great answers.




draft saved


draft discarded














StackExchange.ready(
function () {
StackExchange.openid.initPostLogin('.new-post-login', 'https%3a%2f%2fmath.stackexchange.com%2fquestions%2f3028327%2fare-the-hbl-derivability-conditions-necessary-for-g%25c3%25b6dels-incompleteness-theorem%23new-answer', 'question_page');
}
);

Post as a guest















Required, but never shown





















































Required, but never shown














Required, but never shown












Required, but never shown







Required, but never shown

































Required, but never shown














Required, but never shown












Required, but never shown







Required, but never shown







Popular posts from this blog

Plaza Victoria

In PowerPoint, is there a keyboard shortcut for bulleted / numbered list?

How to put 3 figures in Latex with 2 figures side by side and 1 below these side by side images but in...